¿Existen huecos primos arbitrariamente largos en los que cada número tiene al menos tres factores primos distintos?

Definición: Brecha principal altamente compuesta

Los tres números compuestos entre los primos consecutivos 643 y 647 cada uno tiene al menos tres factores primos distintos. Esta es la primera aparición de la primera brecha de longitud > 1 donde cada número compuesto en el hueco tiene al menos k = 3 factores primos distintos. Llamamos espacio primo entre 643 y 647 como la brecha prima altamente compuesta de orden 3 . Tenemos los espacios primos altamente compuestos para k = 3 , 4 , 5 y 6 como sigue:

  • k = 3 ; pag = 643
  • k = 4 ; pag = 51427
  • k = 5 ; pag = 8083633
  • k = 6 ; pag = 1077940147
  • k = 7 ; pag = 75582271489

Pregunta 1 : ¿Hay infinitos espacios primos de orden altamente compuestos? k 3 ?

Pregunta 2 : Dado k ¿Hay siempre una brecha de orden altamente compuesta? k ?

Una regresión lineal ordinaria entre k y registro pag da un ajuste sorprendentemente fuerte con R 2 0.99915 . Aunque se basa solo en seis puntos de datos, esto sugiere una relación de la forma pag a b k para algunos arreglado a y b .

Definición : Brecha máxima altamente compuesta

La brecha máxima altamente compuesta se define como una brecha principal que es más larga que cualquier brecha anterior y cada compuesto en la brecha tiene al menos 3 factores primos distintos.

Actualización : la brecha más larga que he encontrado es de 75 compuesto consecutivo entre los primos 535473480007 y 535473480083 .

Pregunta 3 : ¿Existen espacios primos arbitrariamente largos en los que cada número compuesto en el espacio tiene al menos tres factores primos distintos?

Actualización : Publicado en MO ya que no tiene respuesta en MSE.

Creo que te refieres a la brecha principal de longitud 3 en la segunda oración. De lo contrario, el número compuesto entre 29 y 31 tiene tres factores primos distintos.
@arbashn Sí, claro. corregido
Tenga en cuenta que para dos números primos consecutivos cualesquiera, pag a y pag b , el intervalo ( pag a 2 , pag b 2 ) contiene al menos una semiprima, pag a pag b , que no tiene al menos tres factores primos distintos. Entonces, los espacios arbitrariamente largos (del tipo en la pregunta) requieren espacios arbitrariamente grandes entre números primos sucesivos. Encontrar ejemplos de tales brechas requerirá trabajar con números muy grandes.
"Término inicial de la primera corrida de exactamente norte números consecutivos con 3 factores primos distintos" está tabulado en oeis.org/A185032 (pero no es exactamente lo que quiere, aún así, las referencias pueden apuntar a algo).

Respuestas (1)

Sí, hay secuencias arbitrariamente largas de números enteros, cada una de las cuales tiene al menos 3 factores primos, y lo mismo es cierto para cualquier k en lugar de 3 .

Dados enteros positivos arbitrarios k y metro , elige números enteros norte 1 , norte 2 , , norte metro con las siguientes propiedades:

  1. Cada norte i tiene al menos k factores primos distintos;
  2. El norte i son pares relativamente primos.

Es obvio que tal secuencia de números enteros existe (simplemente escoja un lote de números primos distintos para cada uno). i , y multiplícalos).

Ahora, usando el Teorema del Resto Chino podemos encontrar un número entero norte tal que para cada i , norte i ( modificación norte i ) , o en otras palabras norte + i 0 ( modificación norte i ) . Por lo tanto, los números norte + 1 , norte + 2 , , norte + metro todos tienen al menos k factores primos distintos.

EDITAR: la pregunta parece exigir aún más que el largo tramo de enteros "muy compuestos" esté reservado por números primos reales. Esta prueba no garantiza eso y, de hecho, sospecho que esto es muy difícil de garantizar.

La pregunta exacta requería que la brecha estuviera entre dos números primos. ¿Hay alguna manera de ajustar su prueba a esto? (Sin embargo, esto prueba lo contrario de lo que traté de mostrar en mi respuesta, por eso eliminé la mía, gracias)
Sí, no está muy claro, pero veo lo que quieres decir. No, creo que sería bastante difícil ajustar mi prueba para manejar esto. Al final de un tramo de números enteros con al menos 3 factores primos, encontraremos números enteros con menos de 3, pero es posible que tengan 2 y no veo la forma de evitarlo.
Por el teorema de Dirichlet sobre progresiones primas, podría haber una manera de asegurar que norte o norte + metro + 1 es primo Pero para obligar a ambos a ser números primos, básicamente necesitas hacer un espacio entre números primos y eso es mucho más difícil que CRT.
Al menos podemos decir que norte y norte + metro + 1 ambos no pueden ser primos si metro incluso...
Probablemente podría acercarse tanto como "un extremo es primo, el otro extremo es primo o semiprimo", utilizando métodos de tamiz. Más allá de eso suena intratable por este enfoque.